PDA

توجه ! این یک نسخه آرشیو شده میباشد و در این حالت شما عکسی را مشاهده نمیکنید برای مشاهده کامل متن و عکسها بر روی لینک مقابل کلیک کنید : سوالاتی پیرامون ریاضی 1 و 2



skyzare
10th December 2011, 05:34 PM
با سلام ...


1-حد این تابع چی میشه ؟



http://latex.codecogs.com/gif.latex?\large&space;\lim_{x\to2}(x-2)^2sin\frac{1}{\sqrt[3]{x-2}}


خوب الان اگه اون sin رو یه تابع کراندار بگیریم حدش صفر میشه . ولی من توی ولفرام نوشتم برای حد چپ و راستش این جوری نوشته . اون complex int چیه ؟ چرا این جوری نوشته ؟



http://www.wolframalpha.com/input/?i=lim+x+to+2+((x-2)^2(sin(1%2F(x-2)^(1%2F3))



================================================== =====


2-این عبارت های زیر چه جوری با هم برابر شده ؟؟؟



http://latex.codecogs.com/gif.latex?\large&space;1^2+2^2+3^2+4^2+....+n^2=\frac{n( n+1)(2n+1)}{6}





http://latex.codecogs.com/gif.latex?\large&space;1^3+3^2+3^3+4^3+....+n^3=\left&space;(\ frac{n(n+1)}{2}&space;\right&space;)^2


================================================== =====




3- میخواستم بدونم این عبارت رادیکالی چه جوری هم ارز با عبارت زیر میشه ؟؟؟؟



http://latex.codecogs.com/gif.latex?\large&space;\lim_{x\to\infty}\sqrt[n]{ax^n+bx^{n-1}+cx^{n-2}+...}\cong&space;\lim_{x\to\infty&space;}(|x+\frac{b}{an}|)( \sqrt[n]{a})

fsohrabi
10th December 2011, 11:20 PM
سوال1
حد عبارت میشه صفر حدی در سینوس(یک به روی بینهایت)که این عبارت حد نداره.
سوال2و3 رو من فقط حفظشون کردم.اثباتشون طولانی بود من هم میونه ی خوبی با اثبات ندارم.
سوال4:
اون عبارت در واقع مجانب مایل تابع هستش که در بینهایت نمودار تابع خیلی به مجانب نزدیک میشه.بنابراین هم ارز هم میشن.
در ضمن من فرمول نویسی با این بلد نیستم!!!!

skyzare
10th December 2011, 11:58 PM
سوال1
حد عبارت میشه صفر حدی در سینوس(یک به روی بینهایت)که این عبارت حد نداره.
سوال2و3 رو من فقط حفظشون کردم.اثباتشون طولانی بود من هم میونه ی خوبی با اثبات ندارم.
سوال4:
اون عبارت در واقع مجانب مایل تابع هستش که در بینهایت نمودار تابع خیلی به مجانب نزدیک میشه.بنابراین هم ارز هم میشن.
در ضمن من فرمول نویسی با این بلد نیستم!!!!

با سلام ...

با تشکر از پاسخ شما ...

ببخشید من رشته ام ریاضی نبوده فنی بودم اگه دیدید سوالم یه خورده الکی هست دیگه ببخشید ! [nishkhand]


1 - در مورد سوال اول ...وقتی از فشردگی حلش میکنیم که جواب صفر به دست میاد ...

میشه یه خورده توضیح بدید دقیقا فرق صفر حدی با مطلق چیه ؟ توی کتابی که میخوندم دیدم ولی توضیح نداده بود . صفر مطلق چه موقع داریم ؟

اون لینک رو دیدید ؟؟؟ نمی دونید چرا این جوری نوشته ؟


2- راستش هر چی بدبختی میکشم از دست همون مطالبی هست که حفظش میکنم . ... [nishkhand]

4-خوب من حالا ببینم میتونم مجانبش رو بدست بیارم یا نه اگه نتونستم مزاحم میشم ! [nishkhand]

==============

در رابطه با فرمول نویسی هم توی کلا هر انجمنی میتونید از این سایت زیر استفاده کنید .


http://www.codecogs.com/latex/eqneditor.php



کافی هست از همون علائم گرافیکی که بالای کادر متن هست استفاده کنید کار کردن باهاش اسون هست یه خورده دست کاری کنید باهاش خودتون متوجه میشد . از طرفی هر چی هم که مینویسید پایینش نمایش میده .

بعد از این که فرمول ریاضی مورد نظر رو نوشتید باید اون پاییین رو روی URL قرار بده بعدش هم ، همون رو کپی کن و آدرسش رو توی گزینه وارد نمودن عکس که بالای پنجره تایپ داخل انجمن هست بذار .

http://s2.picofile.com/file/7166557418/url.png

fsohrabi
11th December 2011, 08:16 AM
درمورد سوال اول عذر میخوام اشتباه نوشتم.دیشب خیلی خسته بودم آخر شبم بود رفتم که بخوابم یادم افتاد.
نیازی به قضیه فشردگی نیست.باید از قضیه صفرو کرانداری استفاده کرد.این قضیه میگه اگر http://latex.codecogs.com/gif.latex?\lim_{x\rightarrow a}f(x)=0 و تابع gدر یک همسایگی محذوف aکراندار باشد آنگاه:
http://latex.codecogs.com/gif.latex?\lim_{x\rightarrow a}f(x)g(x)=0

درست حلش کردین.صفر میشه.
حالا چون تعیین نکرده که 2 از کدوم طرف باید به سمتش xرو میل بدیم وقتی که توی تابع قرارش میدیم صفر مطلق میشه.و برای قسمت sin که میشه سینوس بینهایت که کراندار میشه.(چون سینوس بین -1و1 نوسان میکنه.)پس طبق قضیه صفر و کراندار صفر میشه جواب.
توابع جزء صحیح هم صفر مطلق به وجود میارن.

skyzare
12th December 2011, 06:33 PM
با سلام ...

دوستان میشه یه راهنمایی کنید این حد ها رو چه جوری حل میکنند ؟؟؟؟؟


http://latex.codecogs.com/gif.latex?\large \\\lim _{x\rightarrow +\infty }\frac{[x^2]-[x]^2}{x^2-1}\\\\\\\\ \lim_{x\rightarrow \infty }\frac{\sqrt{x+\sqrt{x+\sqrt{x}}}}{\sqrt{x}}\\\\\\ \\ \lim_{x\rightarrow0 }\frac{\sqrt[4]{x^4+1}-\sqrt{x^2+1}}{x^2}\\\\\\\\ \lim_{x\rightarrow +\infty }\sqrt{x+\sqrt{x+\sqrt{x+\sqrt{x}}}}-\sqrt{x}

abi biya
15th June 2012, 04:59 PM
[negaran]

fsohrabi
15th June 2012, 06:01 PM
با سلام ...

دوستان میشه یه راهنمایی کنید این حد ها رو چه جوری حل میکنند ؟؟؟؟؟


http://latex.codecogs.com/gif.latex?\large \\\lim _{x\rightarrow +\infty }\frac{[x^2]-[x]^2}{x^2-1}\\\\\\\\ \lim_{x\rightarrow \infty }\frac{\sqrt{x+\sqrt{x+\sqrt{x}}}}{\sqrt{x}}\\\\\\ \\ \lim_{x\rightarrow0 }\frac{\sqrt[4]{x^4+1}-\sqrt{x^2+1}}{x^2}\\\\\\\\ \lim_{x\rightarrow +\infty }\sqrt{x+\sqrt{x+\sqrt{x+\sqrt{x}}}}-\sqrt{x}


اولی رو نمیدونم...!اما فک کنم حد نداره...!
دومی هم که میشه بزرگترین درجه صورت به بزرگترین درجه مخرج که فقط رادیکال ایکس صورت میمونه و با مخرج ساده میشه که میشه1...
سومی معلوم نکردین از کدوم طرف...ولی از هوپیتال استفاده کنین.قلم کاغذ جلو دستم نیست حل کنم!!!!پوزش.(ضمنا یه هم ازی هم داره وقت کافی ندارم بنویسم سر فرصت میذارمشون)
چهارمی هم مث دومی فقط اولین رادیکال ایکس رو در نظر میگیریم(چون در بینهایت بزرگترین درجه رو در نظر میگیریم)که از هم که کم شن میشه صفر...

آسد مرتضي
15th June 2012, 10:08 PM
با سلام ...


1-حد این تابع چی میشه ؟



http://latex.codecogs.com/gif.latex?\large&space;\lim_{x\to2}(x-2)^2sin\frac{1}{\sqrt[3]{x-2}}


خوب الان اگه اون sin رو یه تابع کراندار بگیریم حدش صفر میشه . ولی من توی ولفرام نوشتم برای حد چپ و راستش این جوری نوشته . اون complex int چیه ؟ چرا این جوری نوشته ؟



http://www.wolframalpha.com/input/?i=lim+x+to+2+((x-2)^2(sin(1%2F(x-2)^(1%2F3))



================================================== =====


2-این عبارت های زیر چه جوری با هم برابر شده ؟؟؟



http://latex.codecogs.com/gif.latex?\large&space;1^2+2^2+3^2+4^2+....+n^2=\frac{n( n+1)(2n+1)}{6}





http://latex.codecogs.com/gif.latex?\large&space;1^3+3^2+3^3+4^3+....+n^3=\left&space;(\ frac{n(n+1)}{2}&space;\right&space;)^2


================================================== =====




3- میخواستم بدونم این عبارت رادیکالی چه جوری هم ارز با عبارت زیر میشه ؟؟؟؟



http://latex.codecogs.com/gif.latex?\large&space;\lim_{x\to\infty}\sqrt[n]{ax^n+bx^{n-1}+cx^{n-2}+...}\cong&space;\lim_{x\to\infty&space;}(|x+\frac{b}{an}|)( \sqrt[n]{a})

سلام داداش...شرمنده زیاد وقت نداشتم اون دوتای 2 رو که سریع یادم بود میگم
استقرا بلدی؟؟این دو تا با استقرا حل میشه...
اینجوری که میگی برا حالت پایه اسقرا n رو یک میذاری که برقراره.بعد میگی برای n فرض میکنیم درسته میام عبارت رو برای n+1اثبات میکنیم.
http://latex.codecogs.com/gif.latex?\large&space;1^2+2^2+3^2+4^2+....+n^2=\frac{n( n+1)(2n+1)}{6}:فرض میگیریم درسته

http://uc-njavan.ir/images/26qflacthplpg8emv1.gif:حکم جدیدمون

این حکم جدید هم با استفاده از فرض قبلی مون حل میکنیم...اینجوری که به جای از یک تاn عبارت فرض رو گذاشته و به طرف راست تساوی حکم میبریم و عبارت رو باز و ساده میکنیم...و در آخر به یه عبارت درست میرسیم که نشانه اثبات سوال است




http://uc-njavan.ir/images/1ws6vfdydrgkusiozv.gif
http://uc-njavan.ir/images/nzubosvuu6ltnhbp17vh.gif
http://uc-njavan.ir/images/9x8ex1e8yhuuu584z77q.gif

اینم اثباتش....بعدیش هم همینطوریه...شرمنده بعدا تونستم بقیه اش رو هم جواب میدم

saba7
26th July 2012, 03:16 PM
سوال 2 به استقرا ثابت می شه،این فرمول ها همیشه به استقرا جواب میده.

استفاده از تمامی مطالب سایت تنها با ذکر منبع آن به نام سایت علمی نخبگان جوان و ذکر آدرس سایت مجاز است

استفاده از نام و برند نخبگان جوان به هر نحو توسط سایر سایت ها ممنوع بوده و پیگرد قانونی دارد